Step 1: Understand the formula for work done by a variable force.
When a force \(F\) acting on a body is not constant but varies with position \(x\), the work done (\(W\)) by this force in displacing the body from an initial position \(x_1\) to a final position \(x_2\) is given by the definite integral:
\[
W = \int_{x_1}^{x_2} F(x) dx
\]
Step 2: Identify the given force function and displacement limits.
The force acting on the body is \(F(x) = (6x^2 - 4x) \operatorname{N}\).
The initial position is \(x_1 = 2 \operatorname{m}\).
The final position is \(x_2 = 4 \operatorname{m}\).
The mass of the body (10 kg) is provided but is not required to calculate the work done by the force.
Step 3: Set up and evaluate the integral for work done.
Substitute the force function and the limits of integration into the work formula:
\[
W = \int_{2}^{4} (6x^2 - 4x) dx
\]
Now, integrate the expression with respect to \(x\):
\[
W = \left[6 \cdot \frac{x^{2+1}}{2+1} - 4 \cdot \frac{x^{1+1}}{1+1}\right]_{2}^{4}
\]
\[
W = \left[6 \cdot \frac{x^3}{3} - 4 \cdot \frac{x^2}{2}\right]_{2}^{4}
\]
\[
W = \left[2x^3 - 2x^2\right]_{2}^{4}
\]
Now, evaluate the definite integral by substituting the upper limit and subtracting the value obtained by substituting the lower limit:
\[
W = \left(2(4)^3 - 2(4)^2\right) - \left(2(2)^3 - 2(2)^2\right)
\]
Calculate the values:
\[
W = (2(64) - 2(16)) - (2(8) - 2(4))
\]
\[
W = (128 - 32) - (16 - 8)
\]
\[
W = (96) - (8)
\]
\[
W = 88 \operatorname{J}
\]
Step 4: State the final answer.
The work done by the force in displacing the body from \(x = 2 \operatorname{m}\) to \(x = 4 \operatorname{m}\) is 88 J.
The final answer is $\boxed{88 \operatorname{J}}$.